Một số vấn đề về nghiệm của đa thức

Bài viết của thầy Vương Trung Dũng

(Giáo viên chuyên toán trường Phổ thông Năng khiếu)

Trong những kì thi học sinh giỏi các bài toán về đa thức thường xuyên xuất hiện. Tuy nhiên trong chương trình THCS các kiến thức về đa thức chủ yếu dừng lại ở các khái niệm và các phép toán. Do đó khi vừa mới lên lớp 10 các kĩ năng của các em học sinh còn chưa cao. Bài viết này nhằm trình bày một vấn đề nhỏ về nghiệm của đa thức mà nội dung chính là Định lý Bézout và Định lý Viète, đối tượng hướng đến là các em học sinh cuối năm lớp 9 và đầu năm lớp 10.

Trong bài viết này ta kí hiệu $\mathbb{R}[x]$ là tập tất cả các đa thức có hệ số thực.

Cơ sở lý thuyết

Định lý Bézout. Cho $f(x) \in \mathbb{R}[x]$ và $a \in \mathbb{R}$. Số dư khi chia đa thức $f(x)$ cho đa thức $x-a$ là $f(a)$.

Theo thuật toán chia Euclide, tồn tại đa thức $g(x) \in \mathbb{R}[x]$ và số thực $r$ sao cho $$f(x)=(x-a)g(x)+r.$$
Trong đẳng thức trên thay $x=a$ vào hai vế ta được $f(a)=r.$ Từ đó ta có điều phải chứng minh.

Hệ quả 1. Đa thức $f(x)$ có nghiệm $x=a$ khi và chỉ khi $f(x)$ chia hết cho $x-a.$

Hệ quả 2. Nếu $a_1,a_2,…,a_n$ là các nghiệm của $f(x)$ thì $(x-a_1)(x-a_2)…(x-a_n)|f(x)$. Đặc biệt nếu $\deg f=n$ thì $f(x)=c(x-a_1)(x-a_2)…(x-a_n), c\in \mathbb{R}$.

Định lý 2. Một đa thức bậc $n$ có nhiều nhất là $n$ nghiệm. Đặc biệt nếu $\deg f \le n$ có quá $n$ nghiệm thì $f(x) =0.$
Hệ quả 3. Nếu $\deg f<n, \deg g<n$ mà tồn tại $n$ giá trị phân biệt của biến $x$ sao cho $f(x)=g(x)$ thì $f(x)= g(x) .$

Các ví dụ áp dụng.

Ví dụ 1. Biết đa thức $P(x)=x^5+x^2+1$ có 5 nghiệm phân biệt $x_1,x_2,x_3,x_4,x_5$. Đặt $Q(x)=x^2-2$. Tính $Q(x_1)Q(x_2)Q(x_3)Q(x_4)Q(x_5)$.

Lời giải

$P(x)$ có dạng $P(x)=(x-x_1)(x-x_2)(x-x_3)(x-x_4)(x-x_5)$. \

Ta có $$ \prod_{i=1}^{5} Q(x_i)=\prod_{i=1}^{5} (x_i^2-2)=\prod_{i=1}^{5} (\sqrt{2}-x_i) \prod_{i=1}^{5} (-\sqrt{2}-x_i)=P(\sqrt{2})P(-\sqrt{2})=-23. $$

Ví dụ 2. Cho $P(x) \in \mathbb{Z}[x]$ sao cho $|P(a)|=|P(b)|=|P(c)|=1$, với $a,b,c$ là các số nguyên đôi một khác nhau. Chứng minh đa thức $P(x)$ không có nghiệm nguyên.

Lời giải

Giả sử $P(x)$ có nghiệm nguyên $x_0$. Theo định lý Bézout $$ P(x)=(x-x_0)Q(x), \ \ \ \ (1) $$ với $Q(x) \in \mathbb{Z}[x]$. Từ đó suy ra $$ 1=|P(a)|=|a-x_0||Q(a)|. \ \ \ \ (2) $$
Do đó $|a-x_0|=1$, lập luận tương tự ta được $|b-x_0|=|c-x_0|=1$. Như vậy $a-x_0, b-x_0, c-x_0 \in \{-1,1\}$. Theo nguyên lý Dirichlet tồn tại hai trong ba số này bằng nhau từ đó tồn tại hai trong ba số $a,b,c$ bằng nhau, mâu thuẫn. Vậy $P(x)$ không có nghiệm nguyên.

Định lý Viete thuận. Cho đa thức $f \in \mathbb{R}[x]$, trong đó $$f(x)=a_nx^n+a_{n-1}x^{n-1}+…+a_1x+a_0,$$
trong đó $a_i \in \mathbb{R}$ và $a_n \ne 0.$ Giả sử rằng $x_1, x_2,…,x_n$ là các nghiệm (không nhất thiết phân biệt) của $f(x)$. Khi đó ta có

$x_1+x_2+…+x_n=-\dfrac{a_{n-1}}{a_n}$
$x_1x_2+x_1x_3+…+x_{n-1}x_n=\dfrac{a_{n-2}}{a_n}$

$x_1x_2…x_n=(-1)^n \dfrac{a_0}{a_n}$

Chứng minh

Định lý Viète có một ứng dụng rất lớn trong các bài toán về nghiệm của đa thức nhưng chứng minh của nó thì không hề khó. Thật vậy, vì $x_1, x_2,…,x_n$ là các nghiệm của $f$ nên ta có thể viết lại đa thức này dưới dạng $$f(x)=a_n(x-x_1)(x-x_2)…(x-x_n).$$
Khai triển vế phải rồi nhóm về dạng chuẩn tắc, sau đó so sánh hệ số của các số mũ tương ứng ở hai vế ta được điều phải chứng minh.

Lưu ý là định lý Viète vẫn đúng trong trường hợp $f$ không đủ $n$ nghiệm thực, nhưng do đối tượng của bạn đọc nên nội dung bài viết không đề cập đến.

Ví dụ 3. Tìm tất cả các giá trị của $a$ để nghiệm $x_1,x_2,x_3$ của đa thức $x^3-6x^2+ax+a$ thỏa mãn $$(x_1-3)^3+(x_2-3)^3+(x_3-3)^3=0.$$

Lời giải

Đặt $y=x-3$, khi đó $y_1=x_1-3, y_2=x_2-3, y_3=x_3-3$ là nghiệm của đa thức $$ (y+3)^3-6(y+3)^2+a(y+3)+a=y^3+3y^2+(a-9)y+4a-27. $$

Theo định lý Viète $$ \sum_{i=1}^{3} y_i=-3, \sum_{1 \le i<j \le 3} y_iy_j=-9, \prod_{i=1}^{3} y_i=27-4a. $$

Mặt khác theo giả thiết $\sum_{i=1}^{3} y_i^3=0$. Mà $$ \sum_{i=1}^{3} y_i^3=\Big(\sum_{i=1}^{3} y_i\Big)^3-3 \Big(\sum_{1 \le i<j \le 3} y_iy_j\Big)\Big(\sum_{i=1}^{3} y_i \Big)+3 \prod_{i=1}^{3} y_i. $$
Dô đó điều kiện cần và đủ của $a$ là $$ 0=(-3)^3-3(a-9)(-3)+3(27-4a)=-27-3a \Leftrightarrow a=-9. $$

Ví dụ 4. Chứng minh đa thức $P(x)=x^n+2nx^{n-1}+2n^2x^{n-2}+…+2n^{n-1}x+2n$ không thể có đủ $n$ nghiệm thực.

Lời giải
Giả sử $P(x)$ có đủ $n$ nghiệm thực $x_1,x_2,…,x_n$. Theo định lý Viet $$ \sum_{i}x_i=-2n, \sum_{i<j}x_ix_j=2n^2. $$
Khi đó $$ \sum_{i<j}x_ix_j=\dfrac{1}{2}(\sum_{i}x_i)^2-\dfrac{1}{2}\sum_ix_i^2 \le \dfrac{n-1}{2n}(\sum_{i}x_{i})^2=2n(n-1) <2n^2,$$
vô lí. Vậy ta có điều phải chứng minh.

Ta ký hiệu $$\begin{aligned}
\sigma_1 & = \sum_{i=1}^nx_i=-\dfrac{a_{n-1}}{a_n}, \sigma_2=\sum_{1 \le i < j \le n}^nx_ix_j =\dfrac{a_{n-2}}{a_n},…, \
\sigma_k & =\sum_{1 \le i_1 <i_2<…<i_k \le n}x_{i_1}x_{i_2}…x_{i_k}=(-1)^k \dfrac{a_{n-k}}{a_n}
\end{aligned}$$
và gọi $\sigma_k$ là các đa thức đối xứng bậc $k$ của các số $x_1,x_2,…,x_n$.

Định lý Viete đảo. Cho $x_1,x_2,…,x_n \in \mathbb{R}$. Gọi $\sigma_k$ là các đa thức đối xứng bậc $k$ của $n$ số đã cho. Khi đó $x_1,x_2,…,x_n$ là nghiệm của phương trình $$ X^n-\sigma_1X^{n-1}+\sigma_2X^{n-2}+…+(-1)^{n-1}\sigma_{1}X+(-1)^n \sigma_n=0.$$

Ví dụ 5. Gọi $a<b<c$ là 3 nghiệm của phương trình
$$x^3-3x+1=0.$$

a) Tính $A=\dfrac{1-a}{1+a}+\dfrac{1-b}{1+b}+\dfrac{1-c}{1+c};$
b) Tìm một đa thức bậc 3 nhận $a^2-2, b^2-2, c^2-2$ làm nghiệm;

Lời giải
a) Ta có
$$A+3=\dfrac{1-a}{1+a}+1+\dfrac{1-b}{1+b}+1+\dfrac{1-c}{1+c}+1=2\Big(\dfrac{1}{1+a}+\dfrac{1}{1+b}+\dfrac{1}{1+c}\Big).$$
Mặt khác, đặt $x=\dfrac{1}{1+a}$, khi đó $a=\dfrac{1}{x}-1.$ Vì $a^3-3a+1=0$ nên $$\Big(\dfrac{1}{x}-1\Big)^3-3\Big(\dfrac{1}{x}-1\Big)+1=0 \Leftrightarrow 3x^3-3x+1=0.$$
Từ đó suy ra $\dfrac{1}{1+a}, \dfrac{1}{1+b}, \dfrac{1}{1+c}$ là 3 nghiệm của phương trình trên, do đó $$\dfrac{1}{1+a}+\dfrac{1}{1+b}+\dfrac{1}{1+c}=0.$$ Vậy $A=-3.$
b) Theo định lý Viète $a+b+c=0, ab+bc+ca=-3$ và $abc=-1.$ Đặt $P(x)=x^3-3x+1=(x-a)(x-b)(x-c),$
ta có
\begin{eqnarray*}
a^2-2+b^2-2+c^2-2=a^2+b^2+c^2-6=(a+b+c)^2-2(ab+ac+bc)-6=0.
\end{eqnarray*}
Lại có
\begin{eqnarray*}
&&(a^2-2)(b^2-2)+(b^2-2)(c^2-2)+(c^2-2)(a^2-2)\\&=& a^2b^2+b^2c^2+c^2a^2-4(a^2+b^2+c^2)+12\\&=& (ab+bc+ca)^2-2abc(a+b+c) -3.6+12\\ &=&-3.
\end{eqnarray*}
Cuối cùng
\begin{eqnarray*}
&&(a^2-2)(b^2-2)(c^2-2)\\
&=& (\sqrt{2}-a)(\sqrt{2}+a)(\sqrt{2}+c)(-\sqrt{2}-a)(-\sqrt{2}-b)(-\sqrt{2}-c) \\
&=&P(\sqrt{2})P(-\sqrt{2})\\&=&-1.
\end{eqnarray*}
Theo định lý Viète đảo ta có $a^2-2, b^2-2, c^2-2$ là nghiệm của đa thức $x^3-3x+1=0.$

Bài tập có lời giải
Bài 1.  Định $m$ sao cho $F=x^3+y^3+z^3+mxyz$ chia hết cho $x+y+z$.

Lời giải
Xem F là một đa thức theo biến $x.$ Theo giả thiết $F(x) \vdots [x-(-y-z)]$ suy ra $$F(-y-z)=0 \Leftrightarrow (-y-z)^3+y^3+z^3+m(-y-z)yz \Leftrightarrow -yz(y+z)(3+m)=0, $$ với mọi $y,z \in \mathbb{R}$. Từ đó $m=-3.$

 

Bài 2.  (Canada 2001) Cho $P(x)$ là tam thức bậc hai có các hệ số nguyên thỏa mãn đồng thời:
i) Cả hai nghiệm đều nguyên;
ii) Tổng các hệ số là một số nguyên tố;
iii) Tồn tại số nguyên $k$ sao cho $P(k)=55$.

Chứng minh rằng $P(x)$ có một nghiệm bằng 2 và hãy tìm nghiệm còn lại.

Lời giải

Gọi $r_1 \le r_2$ là hai nghiệm. Ta có $P(x)=ax^2+bx+c=a(x-r_1)(x-r_2)$, từ đó $P(1)=a+b+c=a(1-r_1)(1-r_2)=p$ nên $a \in \{\pm 1, \pm p\}$.\

Nếu $a=p$ thì $(1-r_1)(1-r_2)=1$ nên $r_1=r_2=0$ hoặc $r_1=r_2=2$ (mâu thuẫn với (c) ).\

Nếu $a=-p$ thì $(1-r_1)(1-r_2)=-1$ nên $r_1=0, r_2=2$ (cũng mâu thuẫn).\

Vì $P(k)=a(k-r_1)(k-r_2)=-5.11$ nên ta được

$$\begin{cases}
a=1&\\
k-r_1=55&\\
k-r_2=1&
\end{cases} hay \ \begin{cases}
a=1&\\
k-r_1=11&\\
k-r_2=5&
\end{cases}$$

Trong trường hợp đầu tiên ta được $r_2=r_1+54, b=-2r_1-54$ và $c=r_1(r_1+54)$ do đó $r_1^2+52r_1-(53+p)=0$ nên $$ r_1=\frac{-52 \pm \sqrt{52^2+4(53+p)}}{2}= -26 \pm \sqrt{26^2+53+p }=-26 \pm \sqrt{ 27^2+p}.$$

Đặt $h^2=27^2+p \Leftrightarrow p=(h+27)(h-27)$, vì $p$ là nguyên tố nên $h-27=1 \Rightarrow h=28$ nhưng khi dó $p=55$ không là số nguyên tố.\

Trong trường hợp thứ hai $r_2=r_1+6$ nên $b=-2r_1-6$ và $c=r_1(r_1+6)$, do đó $p=10(2r_1+6)+r_1^2+6r_1$ hoặc $$ r_1^2+4r_1-(5+p)=0 \Leftrightarrow r=-1\pm \sqrt{3^2+p}. $$

Đặt $i^2=3^2+p \Leftrightarrow p=(i+3)(i-3), $ vì $p$ là số nguyên tố nên $i=4$ và do đó $p=7 \Rightarrow r_1=2, r_2=8.$

 

Bài 3.  Cho $P(x)=x^n+a_{n-1}x^{n-1}+…+a_1x+a_0$, trong đó $a_k =\pm 1$. Biết $P(x)$ có $n$ nghiệm, chứng minh rằng $n \le 3$.

Lời giải

Giả sử $x_1,…,x_n$ là các nghiệm của $P(x)$. Ta có $\displaystyle \sum_{i=1}^{n} x_i^2=3$ và $\dfrac{1}{x_1}, …, \dfrac{1}{x_n}$ là nghiệm của đa thức $Q(x)=a_0x^n+…+a_{n-1}x+1.$ Ta có $\displaystyle \sum_{i=1}^{n} \dfrac{1}{x_i^2}=3$. Suy ra $$ 9=\Big(\sum_{i=1}^{n} x_i^2 \Big)\Big(\sum_{i=1}^{n} \dfrac{1}{x_i^2}\Big) \ge n^2. $$
Do đó $n \le 3.$

Bài 4.  Cho các số thực $a,b,c$ và phương trình $x^4+4x^3+ax^2+bx+c=0$ có 4 nghiệm thỏa mãn $x_1^{16}+x_2^{16}+x_3^{16}+x_4^{16}=4$. Tìm các nghiệm đó.

Lời giải
Theo định lý Viète ta có $x_1+x_2+x_3+x_4=-4$.\
Áp dụng liên tiếp bất đẳng thức Cauchy Schwarz, ta được
\begin{eqnarray*}16&=&(x_1+x_2+x_3+x_4)^2\\ &\le& 4(x_1^2+x_2^2+x_3^2+x_4^2)\\ &\le& 4\sqrt{4(x_1^4+x_2^4+x_3^4+x_4^4)}\\ &\le& 4 \sqrt{4\sqrt{4(x_1^8+x_2^8+x_3^8+x_4^8)}}\\ &\le& 4 \sqrt{4 \sqrt{4\sqrt{4(x_1^{16}+x_2^{16}+x_3^{16}+x_{4}^{16})}}}=16. \end{eqnarray*}

Dấu "=" xảy ra khi và chỉ khi $x_1=x_2=x_3=x_4=-1$.

Bài 5. (VMO 1991)  Giả sử đa thức $P(x)=x^{10}-10x^9+39x^8+a_7x^7+...+a_1x+a_0$ với các hệ số thực $a_7, ..., a_0$ sao cho đa thức $P(x)$ có 10 nghiệm phân biệt. Chứng minh rằng các nghiệm này thuộc đoạn $[-\frac{5}{2},\frac{9}{2}].$

Lời giải

Gọi $x_1, x_2,…, x_{10}$ là các nghiệm của $P(x)$. Theo định lý Viète ta có
$$ \sum_{i=1}^{10} x_i=10 \ \text{và} \
\sum_{1 \le i <j \le 10} x_ix_j=39.$$

Do đó $$ \Big(\sum_{i=1}^{10} x_i \Big)^2=\sum_{i=1}^{10} x_i^2+2 \sum_{1 \le i<j \le 10} x_ix_j \Rightarrow \sum_{i=1}^{10} x_{i}^2=100-2.39=22. $$

Hơn nữa $$ \sum_{i=1}^{10} (x_i-1)^2=\sum_{i=1}^{10} x_i^2-2 \sum_{i=1}^{10} x_i+10=12 \Rightarrow (x_i-1)^2 \le 12 <(3.5)^2 ,$$
với mọi $i=1,2,…,10.$
Từ đó suy ra điều phải chứng minh.

Bài 6.  Cho các số thực $a,b$ trong đó $a \ne 0.$ Chứng minh rằng tất cả các nghiệm của phương trình $$ax^4+bx^3+x^2+x+1=0$$ không đồng thời là nghiệm thực.

Lời giải
Gọi $\alpha_1, \alpha_2, \alpha_3, \alpha_4$ lần lượt là các nghiệm của phương trình đã cho. Dễ thấy các nghiệm này đều khác 0 và có tích bằng $\dfrac{1}{a}.$ Khi đó nghiệm của phương trình $x^4+x^3+x^2+bx+a=0$ lần lượt là $$\beta_1=\dfrac{1}{\alpha_1}, \beta_2=\dfrac{1}{\alpha_2},\beta_3=\dfrac{1}{\alpha_3},\beta_4=\dfrac{1}{\alpha_4}.$$
Theo định lí Viète $$\sum_{j=1}^{4} \beta_j=-1, \sum_{1 \le j<k \le 4}\beta_j \beta_k=1.$$
Dẫn đến
$$\sum_{j=1}^{4}\beta_j^2=\Big(\sum_{j=1}^{4}\beta_j\Big)^2-2 \Big(\sum_{1 \le j<k \le 4}\beta_j \beta_k\Big)=1-2=-1.$$
Vô lí, bài toán được chứng minh xong.

Bài 7. Giả sử đa thức $ax^3-x^2+bx-1=0$ có 3 nghiệm dương phân biệt. Chứng minh rằng:

a) $0<3ab \le 1;$
b) $b \ge 9a;$
c) $b \ge \sqrt{3}.$

Lời giải
a) Gọi $x_1, x_2, x_3$ là 3 nghiệm của đa thức đã cho. Khi đó theo Định lý Viète, ta có $$x_1+x_2+x_3=\dfrac{1}{a}, x_1x_2+x_1x_3+x_2x_3=\dfrac{b}{a}, x_1x_2x_3=\dfrac{1}{a}.$$
Từ đó suy ra $a>0$ nên $b>0$, dẫn đến $ab>0.$ Từ bất đẳng thức $$(x_1+x_2+x_3)^2 \ge 3(x_1x_2+x_1x_3+x_2x_3)$$ ta được $\dfrac{1}{a^2} \ge 3.\dfrac{b}{a}$ dẫn đến $0 <3ab \le 1.$
b) Vì $(x_1+x_2+x_2)(x_1x_2+x_1x_3+x_2x_3) \ge 9x_1x_2x_3$ nên $\dfrac{b}{a^2} \ge \dfrac{9}{a},$ dẫn đến $b \ge 9a.$
c) Theo bất đẳng thức $(x_1x_2+x_1x_3+x_2x_3)^2 \ge 3x_1x_2x_3(x_1+x_2x+x_3)$ ta được $\dfrac{b^2}{a^2} \ge \dfrac{3}{a^2}$. Dẫn đến $b^2 \ge 3$ và vì $b \ge 0$ nên $b \ge \sqrt{3}.$

Bài 8.  Cho đa thức $x^3+\sqrt{3}(a-1)x^2-6ax+b=0$ có 3 nghiệm thực. Chứng minh rằng $$|b| \le |a+1|^3.$$

Lời giải
Gọi $x_1, x_2, x_3$ là 3 nghiệm của đa thức đã cho, theo định lý Viète $$x_1+x_2+x_3=-\sqrt{3}(a-1), x_1x_2+x_2x_3+x_1x_3=-6a, x_1x_2x_3=-b.$$
Ta có
\begin{eqnarray*}
\sqrt[3]{|b|}= \sqrt[3]{|x_1|.|x_2||x_3|} &\le& \sqrt{\dfrac{x_1^2+x_2^2+x_3^2}{3}} \\&=& \sqrt{\dfrac{(x_1+x_2+x_3)^2-2(x_1x_2+x_2x_3+x_1x_3)}{3}}\\&=& \sqrt{\dfrac{3(1-a)^2+12a}{3}}\\&=& |a+1|.
\end{eqnarray*}
Suy ra $|b| \le |a+1|^3,$ điều phải chứng minh.

 

Bài 9.  [Mathematical Reflections S455] Cho $a,b \in \mathbb{R}$ sao cho tất cả các nghiệm của đa thức
$$P(x)=x^4-x^3+ax+b$$ có 4 nghiệm thực.
a)  Chứng minh rằng $a+ b \ge 0;$
b) Chứng minh rằng $P \Big(-\dfrac{1}{2}\Big) \le \dfrac{3}{16}.$

Lời giải
a) Gọi $x_1, x_2, x_3, x_4$ là 4 nghiệm của đa thức đã cho. Theo định lý Viète ta có
\begin{eqnarray*}
&&x_1+x_2+x_3+x_4=1 \\&& x_1x_2+x_1x_3+x_1x_4+x_2x_3+x_2x_4+x_3x_4=0\\&&-x_1x_2x_3x_4\Big(\dfrac{1}{x_1}+\dfrac{1}{x_2}+\dfrac{1}{x_3}+\dfrac{1}{x_4}\Big)=a\\&&x_1x_2x_3x_4=b.
\end{eqnarray*}
Từ hai phương trình đầu ta được $$x_1^2+x_2^2+x_3^2+x_4^2=1.$$
Theo bất đẳng thức Cauchy-Schwarz
$$1=x_1^2+(x_2^2+x_3^2+x_4^2) \ge x_1^2+\dfrac{1}{3}(x_2+x_3+x_4)^2=x_1^2+(1-x_1)^2.$$
Từ đó ta có $$-\dfrac{1}{2} \le x_1 \le 1.$$
Hoàn toàn tương tự $-\dfrac{1}{2}\le x_2, x_3, x_4 \le 1.$ Khi đó vì $P(x)=(x-1x_1)(x-x_2)(x-x_3)(x-x_4)$ nên dễ thấy $$P(1) \ge 0 \Leftrightarrow a+b \ge 0.$$
b) Bây giờ ta cần chứng minh $$P\Big(-\dfrac{1}{2}\Big) \le \dfrac{3}{16} \Leftrightarrow a \ge 2b.$$
Nếu $b \le 0$ thì từ $a+b \ge 0$ ta suy ra $a \ge 0$ nên hiển nhiên nhiên $a \ge 2b.$ Giả sử $b >0,$ thế thì $x_1x_2x_3x_4 >0$ và do đó ta có
$$a \ge 2b \Leftrightarrow \dfrac{1}{x_1}+\dfrac{1}{x_2}+\dfrac{1}{x_3}+\dfrac{1}{x_4} \le -2. \ \ \ \ (1)$$
Trong trường hợp này phải có hai nghiệm là số dương và hai nghiệm là số âm. Không mất tổng quát giả sử $x_1, x_2>0$ và $x_3, x_4<0$. Vì $-\dfrac{1}{2} \le x_4 \le 1$ nên $2x_4+1 \ge 0, 1-x_4 \ge 0$ và $x_1x_2x_3 <0$. Dẫn đến
\begin{eqnarray*}
x_4^2(1-x_4) \ge x_1x_2x_3(2x_4+1) &\Leftrightarrow& x_4^2(x_1+x_2+x_3) -x_1x_2x_3 \ge 2x_1x_2x_3x_4\\
&\Leftrightarrow& \dfrac{x_4(x_1+x_2+x_3)}{x_1x_2x_3} -\dfrac{1}{x_4} \ge 2\\ &\Leftrightarrow& \dfrac{1}{x_1}+\dfrac{1}{x_2}+\dfrac{1}{x_3}+\dfrac{1}{x_4} \le -2.
\end{eqnarray*}
Bất đẳng thức (1) được chứng minh xong.

Bài 10. Cho số tự nhiên $k>0$ và hai số thực $a, b$ sao cho $x^k + ax + 1$ chia hết cho $x^2 + bx + 1$ và phương trình $x^2 + bx + 1 = 0$ có hai nghiệm. Chứng minh $a(a-b)=0$.

Lời giải
Theo giả thiết tồn tại đa thức $P(x) \in \mathbb{R}[x]$ sao cho $ x^k + ax + 1 = P(x)(x^2 + bx + 1) \ (1).$ Gọi $r_1, r_2$ là hai nghiệm của phương trình $x^2 + bx + 1 = 0$. Khi đó $$(x – r_1)(x – r_2) = x^2 + bx + 1.$$

Theo định lý Viète $\begin{cases}
r_1 + r_2 = -b&\\
r_1r_2 = 1.&
\end{cases}$
Thay vào (1) ta được $$0 = \sum_{i=1}^2 \Big( r_i^k + ar_i + 1 \Big) = r_1^k + r_2^k + a(r_1 + r_2) + 2,$$
suy ra $$r_1^k + r_2^k = -a(r_1 + r_2) – 2 = ab – 2$$ và do đó $$ r_1^k + r_2^k = -a(r_1 + r_2) – 2 = ab – 2.$$
Sử dụng (1) một lần nữa ta được $$a^2r_1r_2 = (r_1^k + 1)(r_2^k + 1) = (r_1r_2)^k + r_1^k + r_2^k + 1.$$
Suy ra $a^2 . 1 = 1^k + (ab – 2) + 1 = ab \Leftrightarrow a(a-b)=0.$

Bài 11.  Cho $P(x) $ là một đa thức hệ số nguyên thỏa mãn các phương trình $P(x)=1, P(x)=2, P(x)=3$ có ít nhất một nghiệm nguyên lần lượt là $x_1, x_2, x_3$.

a) Chứng minh $x_1, x_2, x_3$ là nghiệm nguyên duy nhất của các phương trình trên.
b) Chứng minh rằng phương trình $P(x)=5$ có tối đa một nghiệm nguyên.

Lời giải

a) Vì phương trình $P(x)=2$ nhận $x=x_2$ làm nghiệm nên $$ P(x)=(x-x_2)q(x)+2 \ \ \ \ (1). $$

Vì $P(x)$ là đa thức với hệ số nguyên mà $x_2$ nguyên nên $q(x) \in \mathbb{Z}[x]$. Trong (1) lân lượt thay $x$ bởi $x_1, x_3$ ta được $$ \begin{cases}
1=P(x_1)=(x_1-x_2)q(x_1)+2&\\
3=P(x_3)=(x_3-x_2)q(x_3)+2.&
\end{cases} \Leftrightarrow \begin{cases}
(x_1-x_2)q(x_1)=-1&\\
(x_3-x_2)q(x_3)=1&
\end{cases}.$$
Hơn nữa $x_1 \ne x_3$ nên $\begin{cases}
x_1-x_2=1&\\
x_3-x_2=-1&
\end{cases}$ hoặc $\begin{cases}
x_1-x_2=-1&\\
x_3-x_2=1.&
\end{cases}$\

Trong hai trường hợp ta đều có $x_2=\dfrac{x_1+x_3}{2}$. Giả sử phương trình $P(x)=2$ còn có nghiệm nguyên $x_2′ \ne x_2$ áp dụng lại lập luận trên ta lại có $x_2’=\dfrac{x_1+x_3}{2}=x_2,$ mâu thuẫn. Vậy phương trình này chỉ có một nghiệm nguyên duy nhất là $x_2.$\

Tương tự cho hai phương trình còn lại.

b) Xét phương trình $P(x)=5$.\

Nếu phương trình này không có nghiệm nguyên thì bài toán là hiển nhiên.\

Nếu phương trình này có một nghiệm nguyên $x_5$ thì từ (1) suy ra $$ 5=P(x_5)=(x_5-x_2)q(x_5)+2 \Rightarrow (x_5-x_2)q(x_5)=3. $$

Suy ra $x_5-x_2 \in \{\pm 1, \pm 3\}$.\

Nếu $x_5-x_2=\pm 1$ thì $x_5$ phải trùng với $x_1$ hoặc $x_3$, vô lý.\

Nếu $x_5-x_2= \pm 3$. Vì phương trình $P(x)=3$ nhận $x_3$ làm nghiệm nên $$P(x)=(x-x_3)r(x)+3 \Rightarrow 5=P(x_5)=(x_5-x_3)r(x_5)+3.$$
Để ý rằng $r(x) \in \mathbb{Z}[x]$ nên từ $(x_5-x_3)r(x_5)=2$ nên $x_5-x_3 \in \{\pm 1, \pm 2\}$. Xét hai khả năng:

Trường hợp 1. $\begin{cases}
x_1-x_2=1&\\
x_3-x_2=-1&
\end{cases} \Leftrightarrow \begin{cases}
x_1=1+x_2&\\
x_3=-1+x_2&
\end{cases}$\

– Nếu $x_5-x_2=3 \Rightarrow x_5-x_3=3=(3+x_2)-(-1+x_2)=4$, mâu thuẫn.\

– Nếu $x_5-x_2=-3 \Rightarrow x_5-x_3=(-3+x_2)-(-1+x_2)=-2$, thỏa mãn.\

Tóm lại nếu $\begin{cases}
x_1-x_2=-1&\\
x_3-x_2=1&
\end{cases} \Rightarrow x_5-x_2=-3 \Rightarrow x_5=x_2-3$. Như thế $x_5$ xác định theo $x_1, x_2, x_3$ là duy nhất.\

Trường hợp 2.

Tương tự nếu $$\begin{cases}
x_1-x_2=-1&\\
x_3-x_2=1&
\end{cases} \Rightarrow x_5-x_2=3 \Rightarrow x_5=x_2+3. $$

Như vậy nghiệm nguyên của phương trình này nếu có là duy nhất, bài toán được chứng minh xong.

 

Bài tập rèn luyện

  1. Giả sử đa thức $P(x), Q(x), R(x), S(x) \in \mathbb{R}[x]$ thỏa mãn dẳng thức $$ P(x^5)+xQ(x^5)+x^2R(x^5)=(x^4+x^3+x^2+x+1)S(x).$$
    Chứng minh rằng $P(x)$ chia hết cho $x-1$.
    a) Biết tích của hai trong bốn nghiệm của phương trình $x^4-18x^3+kx^2+200x-2016=0$ là $-32.$ Tìm $k$ .
    b) Biết đa thức $$P(x)=x^n-2nx^{n-1}+2n(n-1)x^{n-2}+...+a_0$$ có $n$ nghiệm thực. Tìm tất cả các nghiệm này.
  2. Giả sử đa thức $P(x)=ax^n-ax^{n-1}+c_2x^{n-2}+...+c_{n-2}x^2-n^2bx+b$ có đúng $n$ nghiệm dương. Chứng minh rằng tất cả các nghiệm này bằng nhau.
    a) Giả sử $x_1, x_2$ là hai trong bốn nghiệm của đa thức $P(x)=x^4+x^3-1$. Chứng minh rằng $x_1x_2$ là nghiệm của đa thức $Q(x)=x^6+x^4+x^3-x^2-1$.
    b) Tìm tất cả các cặp số thực $a,b$ sao cho các đa thức $$P(x)=x^4+2ax^2+4bx+a^2 \ \text{và} \ Q(x)=x^3+ax+b$$ có chung hai nghiệm thực phân biệt.
  3. Cho đa thức $f(x)=3x^3-5x^2+2x-6$ có các nghiệm là $\alpha, \beta, \gamma$. Tính $$T=\Big(\dfrac{1}{\alpha-2}\Big)^2+\Big(\dfrac{1}{\beta-2}\Big)^2+\Big(\dfrac{1}{\gamma-2}\Big)^2.$$
  4. Gọi $r_1, r_2,...,r_7$ là các nghiệm phân biệt của đa thức $P(x)=x^7-7$. Đặt $\displaystyle K=\prod_{1 \le i<j \le 7}(r_i+r_j)$. Tính $K^2.$

 

Tài liệu tham khảo

  1. Phan Huy Khải, Đa thức.
  2. Nguyễn Hữu Điển, Đa thức và ứng dụng.
  3. Titu Andresscu, Navid Safaei, Alessandro Ventullo, Polynomial Problems.
  4. Tạp chí Mathematical Reflections.